Difference between revisions of "2023 AMC 10A Problems/Problem 11"

(removed quote tags)
(redirect)
(Tag: New redirect)
 
(10 intermediate revisions by 5 users not shown)
Line 1: Line 1:
A square of area <math>2</math> is inscribed in a square of area <math>3</math>, creating four congruent triangles, as shown below. What is the ratio of the shorter leg to the longer leg in the shaded right triangle?
+
#redirect[[2023 AMC 12A Problems/Problem 9]]
[asy]
 
size(200);
 
defaultpen(linewidth(0.6pt)+fontsize(10pt));
 
real y = sqrt(3);
 
pair A,B,C,D,E,F,G,H;
 
A = (0,0);
 
B = (0,y);
 
C = (y,y);
 
D = (y,0);
 
E = ((y + 1)/2,y);
 
F = (y, (y - 1)/2);
 
G = ((y - 1)/2, 0);
 
H = (0,(y + 1)/2);
 
fill(H--B--E--cycle, gray);
 
draw(A--B--C--D--cycle);
 
draw(E--F--G--H--cycle);
 
[/asy]
 
<math>\textbf{(A) }\frac15\qquad\textbf{(B) }\frac14\qquad\textbf{(C) }2-\sqrt3\qquad\textbf{(D) }\sqrt3-\sqrt2\qquad\textbf{(E) }\sqrt2-1</math>
 

Latest revision as of 23:45, 9 November 2023